Which fraction is equivalent to -(7/8)

1. -7/8
2. -7/-8
3. -(-7/8)
4. -(7/-8)

Answers

Answer 1

Answer:

A. The others are equivalent to 7/8


Related Questions

Divide 750 into the ratio 2:3​

Answers

Answer:

300 : 450

Step-by-step explanation:

sum the parts of the ratio, 2 + 3 = 5 parts

Divide 750 by 5 to find the value of one part of the ratio

750 ÷ 5 = 150 ← value of 1 part of the ratio , then

2 parts = 2 × 150 = 300

3 parts = 3 × 150 = 450

Then 750 in the ratio 2 : 3 = 300 : 450

The required answer is 750 in the ratio 2 : 3 = 300 : 450.

What is Ratio?

The ratio is defined as a relationship between two quantities, it is expressed as one divided by the other.

We have to divide 750 into the ratio 2:3​

The ratio is given in the question

⇒ 2:3​

We can see that the sum of the parts of the given ratio as

⇒ 2 + 3 = 5 parts

Divide 750 by 5 to determine the value of one part of the ratio

So the value of 1st part of the ratio

⇒ 750 ÷ 5 = 150

Now, 2nd parts = 2 × 150 = 300

And, 3rd parts = 3 × 150 = 450

So, 750 in the ratio 2 : 3 = 300 : 450

Therefore, the required answer is 750 in the ratio 2 : 3 = 300 : 450.

To learn more about the Ratio click here:

brainly.com/question/1504221

#SPJ2

Ewok's favorite candy is a gummy
worm. One gummy worm is 13
inches long: How many gummy
worms will it take to lay them end to
end to have a total length of 45
inches

Answers

Answer:

3.5

Step-by-step explanation:

45 inches divided by 13 inches

gets you 3.4615 but i think you should round up so 3 and a half

Well, if you round the the answer is about 3.5 gummy worms.

This is because 45 divided by 13 equals 3.46153846. That is a VERY long number so you obviously round to the nearest tenth which gives you 3.5. So in the end you would need about 3 and a 1/2 gummy worms to lay them end to end and have a total length of 45 inches. I hope this helps!! (Also that is one really big gummy worm. 13 INCHES?? Hahaha)

Also on this one I’m having trouble

Answers

Answer:

try just 10. without the (-)

it is positive 10 and not negative 10

What is the perimeter of a quadrilateral with vertices at (5,2),
(10,2), (5,5), and (10,5)? PLZZ SHOW WORK AND RIGHT!

Answers

Answer:

16

Step-by-step explanation:

Draw a picture!

Length1 goes from (5, 2) to (10, 2)

since the y coordinate is the same

L1 = (10 - 5) = 5

Length2 goes from (5, 5) to (10, 5)

since the y coordinate is the same

L2 = (10 - 5) = 5

Width1 goes from (5, 2) to (5, 5)

since the x coordinate is the same

W1 = (5 - 2) = 3

Width2 goes from (10, 2) to (10, 5)

since the x coordinate is the same

W2 = (5 - 2) = 3

-----------------------

Perimeter

p = 5 + 5 + 3 + 3

p = 16

The perimeter of quadrilateral is 16 units.

What is quadrilateral?

A closed quadrilateral has four sides, four vertices, and four angles. It is a form of polygon. In order to create it, four non-collinear points are joined. Quadrilateral interior angle sums are always equal to 360 degrees.

Given coordinates,

(5,2), (10,2), (5,5), and (10,5)

let A, B, C, and D be the points of coordinates (5,2), (10,2), (5,5), and (10,5) respectively.

Length1 goes from (5, 2) to (10, 2)

since the y coordinate is the same

AB = (10 - 5) = 5

Length2 goes from (5, 5) to (10, 5)

since the y coordinate is the same

BC = (10 - 5) = 5

Width1 goes from (5, 2) to (5, 5)

since the x coordinate is the same

CD = (5 - 2) = 3

Width2 goes from (10, 2) to (10, 5)

since the x coordinate is the same

DA = (5 - 2) = 3

perimeter = sum of all sides

p = 5 + 5 + 3 + 3 = 16 units

Hence the perimeter is 16 units.

Learn more about quadrilaterals;

https://brainly.com/question/936148

#SPJ2

Complete the table using division to figure out the ratios that are equivalent to 40/28.

? ? 40
? ? 28

Answers

Answer:

[tex]\frac{40}{28} =\frac{20}{14} =\frac{10}{7}[/tex]

Step-by-step explanation:

[tex]\frac{40}{28} =\frac{20}{14} =\frac{10}{7}[/tex]

first divide both the numerator and the dnominator by 2,

then by 4.

Can someone help me with this? Please this would mean so much.

Answers

Answers:

x = angle MLN = 35 degrees

angle FLJ = 55 degrees

=================================================

Explanation:

This problem is fairly tricky if you're not sure what to look for. A slight clue is that they've marked a red point that strangely doesn't have a label on it. It's a fairly small point but it's definitely there if you look closely. While this point's location isn't exactly what we want, we're fairly close. Start at point L and draw a ray through the center point P. Ray LP will intersect the circle at point A, as shown in the diagram below.

From here, draw segments FA and MA. Now notice that inscribed angle LMF = 55 and inscribed angle FAL both subtend the same arc. The term "subtend" basically means "cut off". This arc that the inscribed angles subtend is minor arc FL. A minor arc is where you travel the shorter path around the circle, which indicates its measure is less than 180 degrees.

Since inscribed angles LMF and FAL subtend the same minor arc, this makes the inscribed angles to be congruent.

In short: angle FAL is 55 degrees

----------------------

Segment LA goes through the center P. Through Thales theorem, we know that inscribed angle LFA is 90 degrees. Consequently, we can determine that inscribed angle FLA is 90-55 = 35 degrees.

Segment JL is tangent to the circle, meaning that angle ALJ is 90 degrees. So angle FLJ is 90-35 = 55 degrees.

It's not a coincidence that angle FLJ, angle FAL, and angle LMF are the same measure.

----------------------

We found that angle FAL was 55 degrees. Applying Thales theorem again shows that angle MAF is 90 degrees. Therefore, angle LAM is 90-55 = 35 degrees.

Focus now on triangle LMA. This is also a right triangle (Thales Theorem). The upper acute angle we found was angle LAM = 35, so the lower acute angle is ALM = 55.

Then we can find angle MLN = (angle ALN) - (angle ALM) = 90 - 55 = 35

In short, angle MLN = 35 degrees

Similar to the previous section, it is not a coincidence that angles LFM, LAM and MLN are the same measure.

------------------------

As an alternative, since we know angle FLJ = 55, and angle FLM is 90 degrees, this means...

(angle FLJ)+(angle FLM) + (angle MLN) = 180

55 + 90 + angle MLN = 180

145 + angle MLN = 180

angle MLN = 180 - 145

angle MLN = 35 degrees

HELP ASAP, LINKS AND ABSURD ANSWERS WILL BE REPORTED! I WILL MARK BRAINLIEST!!

Answers

1 1/5. Is the answer

will mark brainliest please help!!

Answers

Answer:

The range was same in both classes

How can you use estimation to justify that the answer $7,875 is reasonable? $1,575 X 5 =p $1,575 X 5 = $7,875. So, p = $7,875.​

Answers

Answer:

p= 1

Step-by-step explanation:

1,575x5 =p  p=1

Suppose that a new employee starts working at ​$7.72 per hour and receives a 5​% raise each year. After time​ t, in​ years, his hourly wage is given by the equation y=$7.72(1.05)t. Find the amount of time after which he will be earning​ $10.00 per hour.

After what amount of time will the employee be earning​ $10.00 per​ hour?

Answers

Answer:

substitute 10 for y (y=10) and solve for t

2.17

Step-by-step explanation:

I think the equation is written wrong because assuming that his wage increases at the end of each year, the beginning of the first year wages should be 7.72, not 0.

recommended correction

y=7.72 + (1.05)t

What is m
A: 19°
B: 68°
C: 112°

Answers

Answer:

m<N=68

Step-by-step explanation:

opposite angles of the parallelogram are equal(or congruent):

4x+36=6x-2

6x-4x=36+2

2x=38

x=38/2=19

the sum of the angles of the parallelogram =360

4x+36=4(19)+36=112 angle P

6x-2=6(19)-2=112 angle M

let Q= N=x

2x+112+112=360

2x=360-224

2x=136

x=136/2=68

Find the volume of each cylinder. Which cylinder has the greater volume? Use 3.14 for π and round your answers to the nearest hundredth.

Two cylinders. The circumference of the base of cylinder A is 3 meters and the height of the cylinder is 5 meters. The circumference of the base of cylinder B is 5 meters and the height of the cylinder is 3 meters.

Answers

Answer:

cylinder A is 141.3 meters

cylinder b is 245.5 meters

Step-by-step explanation:

Cylinder a: 3x3x5x3.14

cylinder B : 5x5x3x3.14

Which problem can be solved with 6divided 1/2

Answers

The answer is d vvvvvvvvvvvvvvvv

Answer:

D

Step-by-step explanation:

6 ounces ÷1/2 ounce slices

What factors greatly affect the climate if you live near the Manila Bay? ​

Answers

Overexploitation of resources, illegal and destructive fishing, habitat destruction, pollution, siltation and sedimentation, uncontrolled development and the conflicting use of limited available resources cause pressures on the bay. Several laws are being implemented HOPE this helps !

Jessica's parents are always telling her to turn off the lights when she leaves her room. The fixture in Jessica's room requires four bulbs that each use 75 watts of
electricity when they're on.
a. Jessica's parents estimate that she leaves the lights on unnecessarily for 2.5 hours per day. How many watt-hours of electricity are used by these bulbs during 2.5
hours?
b. Approximately how many kilowatt-hours of electricity are used in a year to keep these bulbs lit for 2.5 hours per day?
C. At a cost of $0.12 per kilowatt-hour, how much money is wasted per year by keeping these lights on unnecessarily? Round to the nearest dollar.
d. If five million people keep lights on as Jessica does, how much is wasted in unnecessary electric expenses?

Answers

Answer:

Step-by-step explanation:

4*75 = 300 watts ( her light)

a) 2.5*300=750 w/h

b) 365*750 = 273.75  Kw/h

c)   (0.12)*273.75 = $32.85

d) 32.85 * 5,000,000 = $164,250,000.00

Answer:

a.) 750

b.) 273.75

c.) $33.00

d.) $165,000,000.00

2. If curves the length of a circle's diameter were laid out along the circle end to end, how many of them would fit on the circle? (1) more than I but less than 2 (2) more than 2 but less than 3 (3) more than 3 but less than 4 (4) more than 4 but less than 5 by the length of its diameter that ratio is closest to.

If someone could answer this quickly I will mark them brainleast.​

Answers

girl if im being honest i dont know Step-by-step explanation:

Help due soon please help

Answers

Answer:

y=5

Step-by-step explanation:

I’ve attached the steps below. Just keep in mind, when you’re isolating a variable, anything you do to one side of the = must also be applied to the other side.

In ΔOPQ, the measure of ∠Q=90°, OP = 7.2 feet, and PQ = 5.5 feet. Find the measure of ∠P to the nearest degree.

Answers

Answer:

40

Step-by-step explanation:

DeltaMath

Help me with this please

Answers

Answer:

3418.05 (not rounded)

Step-by-step explanation:

Volume = Area * height

V =pi*(8)^2*17

Answer:

i dont know :(

Step-by-step explanation:

23. Assuming the green is flat, what is the radius of the green?


Answers

Answer:

60

Step-by-step explanation:

[tex]r^{2} +32^{2} = (r+8)^{2}[/tex]

[tex]r^{2} +1024=r^{2} +16r + 64[/tex]

[tex]960 = 16r[/tex]

[tex]60=r[/tex]

A solid object is made up of two cubes resting side by side. One cube has an edge length of 5.3 ft, and the other cube has an edge length of 3 ft. What is the volume of the solid object?

Answers

Answer:

2.8

Step-by-step explanation:

Answer:

it's 175.877

Step-by-step explanation:

100% sure

Which of these is part of the prime factorization of 630?

3 with a tiny 2 x 7

5 with a tiny 3

3 with a tiny 3

5 with a tiny 2 x 7​

Answers

Answer:

I don't really get the answer choices but I can show you how I did my work using factor trees.

I put astericks(*these) around the prime #'s(what's in the circles).

      630

         ^

        2.*   315

                 ^

         5. *       63

                        ^

                     3.*   21

                              ^

                            7*.  3*

hope this helps!!

Please HELPPPP I need this right now!

Answers

Answer:

The radius will be half of your diameter so,

since 26 is your diameter half of that is = 13 is your new radius.

Now finding the area:

13^2 = 169

169 x 3.14 = 530.66 cm2 is your area.

Formula of an area of a circle:

πr^2

pi(3.14 or 22/7 depending on whatever they tell you to use for pi) x radius being squared

There are 15 applicants for four jobs: computer programmer, software tester, manager, and systems engineer.



A) Permutation


B) Combination


C) Circular permutation

Answers

Answer:

The answer is "Option A".

Step-by-step explanation:

[tex]\to ^{15}P_{4} = \binom{15}{4}\\\\[/tex]

            [tex]=\frac{15 !}{15-4!}\\\\=\frac{15 !}{11!}\\\\ =\frac{15 \times 14 \times 13 \times 12 \times 11!}{11!}\\\\ = 15 \times 14 \times 13 \times 12 \\\\=32760[/tex]

Answer:

A is the answer the person above is a NERD -_-

Step-by-step explanation:

The area of a circle, in terms of π , is 72 π m2. Find the value of the radius. Give your answer as a simplified surd.

Answers

Answer:

R=3rad2

Step-by-step explanation:

Area of circle: pi r^2

72pi=pir^2

r^2=72

r=6rad2

Rick wants to use multiplication to solve {k} = 7 \div \dfrac13k=7÷ 3 1 ​ k, equals, 7, divided by, start fraction, 1, divided by, 3, end fraction. Which multiplication equation can Rick use? Choose 1 answer: Choose 1 answer: (Choice A) A \dfrac13\times{k} =7 3 1 ​ ×k=7start fraction, 1, divided by, 3, end fraction, times, k, equals, 7 (Choice B) B \dfrac13={k}\times7 3 1 ​ =k×7start fraction, 1, divided by, 3, end fraction, equals, k, times, 7 (Choice C) C \dfrac13\times7 = {k} 3 1 ​ ×7=kstart fraction, 1, divided by, 3, end fraction, times, 7, equals, k

Answers

Answer: (a)

Step-by-step explanation:

Given

Equation is

[tex]k=\dfrac{7}{\frac{1}{3}}[/tex]

Write the cross-multiplication of the given equation i.e.

[tex]\Rightarrow \dfrac{1}{3}\times k=7[/tex]

This corresponds to the option (a).

Answer:

C

Step-by-step explanation:

ITS C BECAUSE I WASTED A MISTAKE BC OF THE OTHER GUY ON TOP

DONT USE HIS

JUST USE MINE

DONT NEED AN EXPLANATION NOOOOB

please help me with geometry questions ​

Answers

Answer:

[tex]\angle NPM[/tex]

Step-by-step explanation:

Given

Circle of center P

Polygon JKLMN

Required

The central angle of JKLMN

From the question, we have that: the polygon and the circle have the same center (P).

This means that any central angle must pass through P

By this, PMN and NJK are incorrect

QPK divides the polygon and point Q is not on the vertex of the polygon.

By this, QPK is also incorrect

The final option NPM is correct, because P is at the center and points N and M are at the vertices of the polygon

Select the correct comparison.
Set A
Set B
0
2 A
68
2 4 6 8 10
O.
A. The typical value is greater in set A. The spread is greater in set B.
B. The typical value and the spread are both greater in set B.
C. The typical value is greater in set B. The spread is greater in set A.
ОО
D. The typical value and the spread are both greater in set A.

Answers

Answer:

C

Step-by-step explanation:

Ms. Pacheco wrote the following expression. [7 x (4 + 11)] - 9 What is the value of the expression?

Answers

Answer: 105x-9

Step-by-step explanation:

Sorry love don’t feel like typing the explanation! :(

What are the graph coordinates of the vertex of the graph function y=x^2+6x-11

Answers

Answer:

(3-2) hope it helps sorry if wrong it should be right

Answer:

(-3, -20)

Step-by-step explanation:

Other Questions
Can you help me plz with this work What is the midpoint of AB when A (-2, 5) and B (4, -5)? PLEASE HELP ME! IS THIS CORRECT? a bus company operates in dubai has the following operation times.calculate the total number of hours that the bus company operates in one week. Question 4 (10 points)If a sollition has a pOH of 5.2 the [OH-] of the solution is6x 10-6 MOb6.3 x 10-6 MOc1.58 x 10-5MOd2x10-5M What is the main idea of "A dead woman's secret" PLZ answer as soon as possible :) In an aquarium, there are 6 large fish and 8 small fish. Half of the small fish are red.One fish is selected at random. Find the probability that it is a small, red fish.Write your answer as a fraction in simplest form. 2Which point on this number line best represents 10?point Apoint Bpoint C point D 1. Which of the angles below are supplementary? Someone please help me with this Alonzo runs each lap in 6 minutes. He will run at most 30 minutes today. What are the possible numbers of laps he will run today?Use n for the number of laps he will run today.Write your answer as an inequality solved for n. please answer this as quick as possible. Which options are available when exporting a table definition and data? Check all that apply Find the mode 9,10,15,17,15,14,37,45,42,13 need help ASAP !! A line segment has endpoints(3,14) and (3,22) which of the following statements is true? Following _______ & _________ makes life disciplined. Which one of these sentences uses grammatical punctuation? Jax hadn't as far as I'm aware already seen the movie before we went to see it together. 1. Find the measure of arc ABCplease show ur work Help ASAP will give brainly 9th grade math poop I don't know what to do-